Familiar, isn't it?

Algebra Level 5

x 2 + y 2 + x y + y 2 + z 2 + y z + x 2 + z 2 + x z \large \sqrt{x^2+y^2+xy}+\sqrt{y^2+z^2+yz}+\sqrt{x^2+z^2+xz} If x , y x,y and z z are positive reals satisfying x y + y z + x z 9 x y z xy+yz+xz\leq 9xyz , minimize the expression above to 2 decimal places.


The answer is 1.73.

This section requires Javascript.
You are seeing this because something didn't load right. We suggest you, (a) try refreshing the page, (b) enabling javascript if it is disabled on your browser and, finally, (c) loading the non-javascript version of this page . We're sorry about the hassle.

4 solutions

We can also use AM-GM inequality.

Since x 2 + y 2 + x y 3 x y x^2+y^2+xy \ge 3xy and similar for the other two expressions, we have:

x 2 + y 2 + x y + y 2 + z 2 + y z + z 2 + x 2 + z x 3 x y + 3 y z + 3 z x 3 ( 3 x y z 3 ) = 3 3 x y z 3 \begin{aligned} \sqrt{\color{#3D99F6}{x^2+y^2+xy}} + \sqrt{y^2+z^2+yz} + \sqrt{z^2+x^2+zx} & \ge \sqrt{\color{#3D99F6}{3xy}} + \sqrt{3yz} + \sqrt{3zx} \\ & \ge \sqrt{3}\left(3\sqrt [3]{xyz} \right) = 3\sqrt{3}\sqrt [3]{xyz} \end{aligned}

Also,

x y + y z + z x 9 x y z But x y + y z + z x 3 ( x y z 3 ) 2 9 x y z 3 ( x y z 3 ) 2 x y z 3 1 3 \begin{aligned} xy + yz + zx & \le 9xyz \\ \text{But} \quad xy + yz + zx & \ge 3 \left( \sqrt [3] {xyz} \right)^2 \\ \Rightarrow 9xyz & \ge 3 \left( \sqrt [3] {xyz} \right)^2 \\ \Rightarrow \sqrt [3] {xyz} & \ge \frac{1}{3} \end{aligned}

Therefore,

x 2 + y 2 + x y + y 2 + z 2 + y z + z 2 + x 2 + z x 3 3 x y z 3 3 3 × 1 3 = 3 \begin{aligned} \sqrt{x^2+y^2+xy} + \sqrt{y^2+z^2+yz} + \sqrt{z^2+x^2+zx} & \ge 3\sqrt{3}\sqrt [3]{xyz} \\ & \ge 3 \sqrt{3} \times \frac{1}{3} = \boxed{\sqrt{3}} \end{aligned}

(Being pedantic, because this is something that most people miss out.)

You have shown that a lower bound exists. Why is that the maximum lower bound?

Calvin Lin Staff - 5 years, 3 months ago
P C
Feb 6, 2016

From the condition, we get 9 1 x + 1 y + 1 z 9 x + y + z x + y + z 1 9\geq\frac{1}{x}+\frac{1}{y}+\frac{1}{z}\geq\frac{9}{x+y+z}\Rightarrow x+y+z\geq 1 The problem can be written as A = ( x + y 2 ) 2 + 3 y 2 4 + ( y + z 2 ) 2 + 3 z 2 4 + ( z + x 2 ) 2 + 3 x 2 4 A= \sqrt{\bigg(x+\frac{y}{2}\bigg)^2+\frac{3y^2}{4}}+\sqrt{\bigg(y+\frac{z}{2}\bigg)^2+\frac{3z^2}{4}}+\sqrt{\bigg(z+\frac{x}{2}\bigg)^2+\frac{3x^2}{4}} So using Minkowski inequality we get A 3 ( x + y + z ) 3 1.73 A\geq\sqrt{3}(x+y+z)\geq\sqrt{3}\approx 1.73 The equality holds when x = y = z = 1 3 x=y=z=\frac{1}{3}

Just to clarify my use of the Minkowski's Inequality, that is A ( x + y 2 + y + z 2 + z + x 2 ) 2 + ( 3 y 2 + 3 z 2 + 3 x 2 ) 2 \displaystyle A \ge \sqrt{(x + \frac{y}{2} + y + \frac{z}{2} + z + \frac{x}{2})^2 + (\frac{\sqrt{3}y}{2} + \frac{\sqrt{3}z}{2} + \frac{\sqrt{3}x}{2})^2} = 9 + 3 4 ( x + y + z ) 2 = 3 ( x + y + z ) . \displaystyle = \sqrt{\frac{9 + 3}{4}(x + y + z)^2} = \sqrt{3}(x + y + z). Am I right?

Reineir Duran - 5 years, 4 months ago

Log in to reply

yes, you're right

P C - 5 years, 4 months ago
Zk Lin
Feb 11, 2016

Here is a solution using a combination of both AM-GM and Cauchy-Schwarz inequality.

Note that 9 x y z x y + y z + x z 3 ( x y z ) 2 3 9xyz \geq xy+yz+xz \geq 3(xyz)^{\frac{2}{3}} by AM-GM inequality.

Simplifying, we get

( x y z ) 1 3 1 3 (xyz)^{\frac{1}{3}} \geq \frac{1}{3}

Note that

c y c x 2 + y 2 + x y \displaystyle \sum_{cyc} \sqrt{{x^{2}+y^{2}+xy}}

= c y c ( x 2 + y 2 + x y ) ( 1 3 + 1 3 + 1 3 ) =\displaystyle \sum_{cyc} \sqrt{{(x^{2}+y^{2}+xy)(\frac{1}{3}+\frac{1}{3}+\frac{1}{3})}}

c y c ( 1 3 . x + 1 3 . y + 1 3 . x y ) \geq \displaystyle \sum_{cyc} (\frac{1}{\sqrt{3}}.x+\frac{1}{\sqrt{3}}.y+\frac{1}{\sqrt{3}}.\sqrt{xy}) by Cauchy-Schwarz inequality

= 2 3 ( x + y + z ) + 1 3 . ( x y + y z + x z ) =\frac{2}{\sqrt{3}}(x+y+z)+\frac{1}{\sqrt{3}}.(\sqrt{xy}+\sqrt{yz}+\sqrt{xz})

2 3 . 3. ( x y z ) 1 3 + 1 3 . 3. ( x y z ) 1 3 \geq \frac{2}{\sqrt{3}}.3.(xyz)^{\frac{1}{3}}+ \frac{1}{\sqrt{3}}.3.(xyz)^{\frac{1}{3}} by AM-GM inequality,

2 3 . 3. 1 3 + 1 3 . 3. 1 3 \geq \frac{2}{\sqrt{3}}.3.\frac{1}{3}+ \frac{1}{\sqrt{3}}.3.\frac{1}{3} (from above)

= 3 3 =\frac{3}{\sqrt{3}}

= 3 =\boxed{\sqrt{3}}

Equality holds when x 2 = y 2 = x y , y 2 = z 2 = y z , z 2 = x 2 = x z x^{2}=y^{2}=xy, y^{2}=z^{2}=yz, z^{2}=x^{2}=xz (condition for Cauchy-Schwarz inequality to hold, which simplifies to x = y = z = 1 3 x=y=z=\frac{1}{3} ). This happens to be the same condition for AM-GM inequality to hold, which we apply multiple times in our solution. We check also that x y + y z + x z 9 x y z xy+yz+xz \leq 9xyz when x = y = z = 1 3 x=y=z=\frac{1}{3}

Moderator note:

For completeness, you should check that the equality condition satisfies the initial condition of x y + y z + x z 9 x y z xy+yz+xz\leq 9xyz . Otherwise, equality need not hold.

For completeness, you should check that the equality condition satisfies the initial condition of x y + y z + x z 9 x y z xy+yz+xz\leq 9xyz . Otherwise, equality need not hold.

Calvin Lin Staff - 5 years, 3 months ago

Log in to reply

Agreed. I have edited this into my solution.

ZK LIn - 5 years, 3 months ago
Son Nguyen
Feb 7, 2016

We can apply vector into this problem.

Can you explain in detail? Thanks!

Calvin Lin Staff - 5 years, 3 months ago

0 pending reports

×

Problem Loading...

Note Loading...

Set Loading...